LSAT and Law School Admissions Forum

Get expert LSAT preparation and law school admissions advice from PowerScore Test Preparation.

 Administrator
PowerScore Staff
  • PowerScore Staff
  • Posts: 8916
  • Joined: Feb 02, 2011
|
#73475
Complete Question Explanation
(The complete setup for this game can be found here: https://forum.powerscore.com/lsat/viewtopic.php?t=3901)

The correct answer choice is (A)

This List question can be solved by applying the proper List question technique of applying one rule to all contending answer choices until the four incorrect answers are eliminated.

Answer choice (B) is incorrect because if neither L or T perform on the north stage, then they must both perform on the south stage, a violation of the fourth rule.

Answer choice (C) is incorrect because if H performs at 6, the second rule is violated.

Answer choice (D) is incorrect because both L and T perform on the same stage, a violation of the fourth rule.

Answer choice (E) is incorrect because the PQ ordering violates the last rule.

Thus, answer choice (A) is proven correct by process of elimination.

Get the most out of your LSAT Prep Plus subscription.

Analyze and track your performance with our Testing and Analytics Package.